Esercizi misti limiti 3

Esercizi misti sui Limiti

Home » Esercizi misti limiti 3

Richiamiamo di seguito solo i principali risultati che verranno utilizzati per la risoluzione degli esercizi. Per i richiami teorici più completi si rimanda alle dispense di teoria sui limiti notevoli , alla dispensa sui simboli di Landau e a quella sulle forme indeterminate.

Teorema 1 – Teorema dei carabinieri. 

Siano f,g, h \colon A \subseteq \mathbb{R} \to \mathbb{R} e sia x_0 \in \mathbb{R} \cup \{\pm\infty\} un punto di accumulazione per A. Si assuma che

    \[\lim_{x \to x_0} f(x) = \lim_{x \to x_0} h(x) \eqqcolon \ell \in \mathbb{R} \cup \{\pm\infty\}\]

e che in un intorno di x_0, denotato con I(x_0) si abbia

    \[f(x) \le g(x) \le h(x), \qquad \forall x \in I(x_0)\setminus\{x_0\}.\]

Allora

    \[\lim_{x \to x_0} g(x) = \ell.\]

 

Teorema 2. 

Siano f, g\colon A \subseteq \mathbb{R}\to \mathbb{R}, sia x_0 \in \mathbb{R} \cup \{\pm \infty\} un punto di accumulazione per A. Si assuma che

    \[\exists \lim\limits_{x \to x_0} f(x) =: \ell_1, \qquad \exists \lim\limits_{x \to x_0} g(x) =: \ell_2,\]

allora, ogni qualvolta l’espressione a destra non è un forma indeterminata, si ha:

    \[\begin{aligned} \exists \; \lim\limits_{x \to x_0}(f\pm g)(x) & =\ell_1 \pm \ell_2 \\ \exists \; \lim\limits_{x \to x_0}(f\cdot g)(x) & = \ell_1 \cdot \ell_2, \end{aligned}\]

Se x_0 è un punto di accumulazione per \{x \in A \colon g(x) \neq 0\}, allora si ha:

    \[\exists \; \lim\limits_{x \to x_0} \left( \dfrac{f}{g}\right)(x) = \dfrac{\ell_1}{\ell_2},\]

ogni qualvolta l’espressione a destra esiste e non è una forma indeterminata.

 

Teorema 3 – Teorema di sostituzione. 

Sia f\colon A \subseteq \mathbb{R}\to \mathbb{R} e sia x_0 \in \mathbb{R}\cup \{\pm \infty\}. Si assuma che

    \[\exists \lim\limits_{x \to x_0} f(x) = \ell \in \mathbb{R}\cup \{\pm \infty\}.\]

Sia I(\ell) un intorno di \ell e sia g \colon I(\ell) \to \mathbb{R} tale che

  1. se \ell \in \mathbb{R}, g è continua in \ell;
  2. se \ell = \pm \infty, allora esiste \lim\limits_{y \to \ell}g(y).

Allora,

    \[\lim\limits_{x \to x_0} g(f(x)) = \lim\limits_{y \to \ell}g(y).\]

 

 

Testi degli esercizi

Esercizio 3  (\bigstar\bigstar\largewhitestar\largewhitestar\largewhitestar). Calcolare, se esistono, i seguenti limiti:

    \[\begin{aligned} &1.\quad \lim\limits_{x\rightarrow+\infty}\left(\dfrac{x^2-1}{x^2+1}\right)^x;\\[10pt] & 2.\quad \lim\limits_{x\rightarrow e}(\log x)^{\frac{1}{x-e}};\\[10pt] &3.\quad \lim\limits_{x\rightarrow 0}\dfrac{\sin(x\cos x)}{x};\\[10pt] &4.\quad \lim\limits_{x\rightarrow 0}\dfrac{\log(\cos x)}{x^2};\\[10pt] &5.\quad \lim\limits_{x\rightarrow 0}\dfrac{\sinh x}{x}.\end{aligned}\]

 
Svolgimento.
1. Abbiamo

    \[\begin{aligned} 	\lim_{x\rightarrow+\infty}\left(\frac{x^2-1}{x^2+1}\right)^x&=\lim_{x\rightarrow+\infty}\left(\frac{x^2+1-2}{x^2+1}\right)^x\\ &=	\lim_{x\rightarrow+\infty}\left(1+\frac{-2}{x^2+1}\right)^x\\ &=\lim_{x\rightarrow+\infty}\left(\left(1-\frac{2}{x^2+1}\right)^{\frac{x^2+1}{-2}}\right)^{\frac{-2x}{x^2+1}}=e^0=1. \end{aligned}\]

Si noti che si è utilizzato il limite notevole

    \[\lim_{t \to -\infty} \left(1+\dfrac{1}{t}\right)^t=e\]

con la sostituzione t=- \dfrac{x^2+1}{2}, per il teorema 3. Inoltre si è sfruttato il fatto che

    \[\lim_{x \to +\infty} \frac{2x}{x^2+1}= 0.\]

Infine, si è usata la continuità della funzione esponenziale:

    \[\text{se }\lim_{x \to x_0}f(x)=y_0, \text{ allora } \lim_{x \to x_0} e^{f(x)}=e^y_0.\]

 
2. Si osservi che

    \[(\log x)^{\frac{1}{x-e}}=\exp\left(\log\left(\left(\log x\right)^{\frac{1}{x-e}}\right)\right)=\exp\left(\frac{\log(\log x)}{x-e}\right).\]

Per il teorema 3, ponendo \log x=1+t, si ha che x=e^{1+t}=e\cdot e^t, per cui t\rightarrow 0 se x\rightarrow e. Si ha:

    \[\begin{aligned} 	\lim_{x\rightarrow e}\exp\left(\frac{\log(\log x)}{x-e}\right)&=\lim_{t\rightarrow 0}\exp\left(\frac{\log(1+t)}{e(e^t-1)}\right)\\ 	&=\lim_{t\rightarrow 0}\exp\left(\frac{1}{e}\cdot\frac{\log(1+t)}{t}\cdot\frac{t}{e^t-1}\right)\\ 	&=\exp\left(\frac{1}{e}\right)=e^{1/e}, 	\end{aligned}\]

dove si è utilizzata la continuità della funzione esponenziale, i limiti notevoli del logaritmo e dell’esponenziale e il teorema 2.
 
3.Si ha

    \[\lim_{x\rightarrow 0}\frac{\sin(x\cos x)}{x}=\lim_{x\rightarrow 0}\frac{\sin(x\cos x)}{x\cos x}\cdot\cos x=1\cdot 1=1,\]

dove nel penultimo passaggio si è utilizzato il teorema 3 ponendo t = x \cos x per applicare il limite notevole del seno.
 
4. Si ha

    \[\lim_{x\rightarrow 0}\frac{\log(\cos x)}{x^2}=\lim_{x\rightarrow 0}\frac{\log(\cos x)}{1-\cos x}\cdot\frac{1-\cos x}{x^2}.\]

Per il teorema 3, posto \cos x=t+1, se x\rightarrow 0 allora t\rightarrow 0, quindi

    \[\lim_{x\rightarrow 0}\lim_{t\rightarrow 0}\frac{\log(t+1)}{-t}=-1.\]

Poiché

    \[\lim_{x \to 0}\frac{1-\cos x}{x^2} = \dfrac{1}{2},\]

per il teorema 2 il limite è -\frac{1}{2}.
 
5. Per definizione si seno iperbolico si ha:

    \[\sinh x=\frac{e^x-e^{-x}}{2},\qquad \forall x \in \mathbb{R}.\]

e quindi

    \[\lim_{x\rightarrow 0}\frac{\sinh x}{x}=\lim_{x\rightarrow 0}\frac{e^x-e^{-x}}{2x}=\frac{1}{2}\cdot\lim_{x\rightarrow 0}\left(\frac{e^x-1}{x}+\frac{e^{-x}-1}{-x}\right)=\frac{1}{2}\cdot 2=1,\]

dove nella parentesi si è sommato e sottratto 1 per poi dividere la frazione, al fine di ottenere il limite notevole della funzione esponenziale.

error: Il contenuto è protetto!!